Thread Rating:
  • 0 Vote(s) - 0 Average
  • 1
  • 2
  • 3
  • 4
  • 5
Q of the Day - vanco
#21
as pul edema is included in un-stability???

vancooooooooooooooooooo
Reply
#22
senility,PLS. read my above explanation regarding B, this is by current 2008,
doc649 , you are right about using TEEE with Cardiovert after 3 days but :1st. it is byeond step2 exam, and you have to try to control the rate with other methods before considering to sedate your patient for the TEEE.
anyway guys , this is in the real life, in the exam sometimes we don'nt know what they realy want us to answer
Reply
#23
A.
Reply
#24
i will go for A. whats the ans vanco
Reply
#25
B...
Reply
#26
i would go with c.patint i dnt think so is unstable.
Reply
#27
c c c.
unstable(pul edema)---->TEE--->DC cadioversion---->then anti coagulation--->then ccbs/digoxin to control sinus rhythm
Reply
#28
Answer:

(B) Diltiazem

Explanation:

This patient has a diastolic murmur and an opening snap consistent with mitral stenosis. All the therapies described may be useful in the management of mitral stenosis. As is often the case on board tests, all the answers are partially correct. The initial step is to relieve this patient's symptoms by controlling the heart rate. Ventricular filling is impaired by mitral stenosis. The ventricle fills during diastole. The rapid rate of atrial fibrillation shortens diastolic filling time and causes the symptoms. The only therapy listed in the answer choices that controls heart rate is diltiazem. Although furosemide will decompress the lungs, it will not slow the heart rate. And although he may eventually need balloon valvotomy, this would not be done before the heart rate has been controlled. Coumadin will eventually be needed; worrying about a clot that might form in a year is not as important as controlling the symptoms of dyspnea now. It is unlikely that anything found on an echocardiogram will make you not control the rate. The echocardiogram is needed but will not change the initial management. Electrical cardioversion is not indicated for several reasons. First, he is not acutely unstable. The dyspnea is on exertion, not right now. Second, with mitral stenosis and what is surely an accompanying left atrial dilation, he will probably revert back to atrial fibrillation. The more abnormal the atrium is anatomically, the harder it is to successfully cardiovert. Finally, you would not want to cardiovert atrial fibrillation in a patient with three days of symptoms without either a transesophageal echo to exclude a clot or without having given three weeks of anticoagulation prior to the cardioversion.
Reply
#29
I wanted to post it here that its a very nice qs with very nice explanation..but some how posted in ur imp qs thread..anyways thats also a good one!!
Reply
« Next Oldest | Next Newest »


Forum Jump: